Quadrilateral ABCD has vertices A(–1, –2), B(–1, 3), C(4, 3) and D(4, –2). It’s dilated by a factor of 2 with the center of dilation at the origin. What are the coordinates of the resulting quadrilateral A’B’C’D

Answers

Answer 1

9514 1404 393

Answer:

A'(-2, -4)B'(-2, 6)C'(8, 6)D'(8, -4)

Step-by-step explanation:

Dilation about the origin multiplies each coordinate value by the dilation factor.

  A' = 2A = 2(-1, -2) = (-2, -4)

  B' = 2B = 2(-1, 3) = (-2, 6)

 C' = 2C = 2(4, 3) = (8, 6)

  D' = 2D = 2(4, -2) = (8, -4)


Related Questions

How would I find the area of this?

Answers

Answer:

Read below v

Step-by-step explanation:

First you would find the area of the semi circle by turning it into a circle and then dividing your answer by two. Then you would find the area of the triangles.

Answer:

26.13 [tex]ft^{2}[/tex]

Step-by-step explanation:

Half circle :

Area of a circle = [tex]\pi r^{2}[/tex]

A = 3.14 ([tex]3^{2}[/tex]) = 9(3.14) = 28.26

That's a full circle. We want half. Divide by 2 = 14.13

Triangles. Both are same.

A = [tex]\frac{(b)(h)}{2}[/tex] = [tex]\frac{(3)(4)}{2}[/tex] = [tex]\frac{12}{2}[/tex] = 6

There are two of them, so area of triangles is 12

12 + 14.13 = 26.13

A rocket is fired upward with an initial velocity v of 80 meters per second. The quadratic function S(t)=−5t2+80t can be used to find the height s of the​ rocket, in​ meters, at any time t in seconds. Find the height of the rocket 8 seconds after it takes off. During the course of its​ flight, after how many seconds will the rocket be at a height of 270 ​meters?

Answers

9514 1404 393

Answer:

  a) 320 m

  b) 4.838 seconds, or 11.162 seconds

Step-by-step explanation:

a) S(8) = -5(8^2) +80(8) = -320 +640 = 320

The height 8 seconds after takeoff will be 320 meters.

__

b) S(t) = 270

  270 = -5t^2 +80t

  -54 = t^2 -16t . . . . . . divide by -5

  10 = t^2 -16t +64 . . . . add 64

  ±√10 = t -8 . . . . . . . . . . take the square root

  t = 8 ±√10 ≈ {4.838, 11.162}

The rocket will be at a height of 270 meters at 4.838 and 11.162 seconds after takeoff.

Which equation is equivalent to
1
3
= 27X+22
O 3*-3-3x+2
O 3* = 33x+6
O 3-* - 33x+2
O 3-* = 33x+6

Answers

Answer:

[tex]3^{-x} = 3^{3x + 6}[/tex]

Step-by-step explanation:

Required

Which is equivalent to [tex](\frac{1}{3})^x = 27^{x + 2}[/tex]

Express base as 3

[tex](3^{-1})^x = 3^3^{x + 2}[/tex]

Open brackets

[tex]3^{-x} = 3^{3x + 6}[/tex]

Abu is trying to decide which pet–sitting service he wants to use . Your Pets charges a $15 fee, plus $1 .75 per hour . Sit Pets charges an $11 fee, plus $2 .25 per hour . At how many hours will both services ...

Answers

Answer:I did not see the entire question but is assuming the question is asking how many hours for both services to cost the same.

Your Pets Cost =15+1.75x

Sit Pets Cost =11+2.25x

set both equations equal to each other

15+1.75x=11+2.25x

15-11 = (2.25-1.75)x

4=0.5x

x=8

Step-by-step explanation:

821) The integon which is 15 more than - 55 is

Answers

Answer:

-40

Step-by-step explanation:

-55 + 15 = x

-40 =x

Solve the inequality (help please)

Answers

Answer:

v<1 23/25

Step-by-step explanation:

The inequality simplifies to 48/25, which is equivalent to 1 23/25.

900 d 180 nd the SP when: CP = Rs. 950, gain = 6 1%4 ​

Answers

Respuesta:

????????????,,,???

Step-by-step explanation:

THERE ARE STEPS TO THIS

If someone knows the answer that will be greatly appreciated :)

Answers

[tex]a. \: 8 = 2 + 2a \: and \: 3 = a[/tex]

Step-by-step explanation:

Set up equation as we know both weights are equal.Subtract two from both sides and divide by 2 to isolate the variable a.3 = a

Write the equation of the line in point-slope form given the information:

Slope = -1/5

Y intercept = -3

Answers

point slope form:

y + 3= -1/5 x (x+0)

Step-by-step explanation:

-use the point slope form equation: y - y1 = m (x - x1).

-using the given information we know that m = -1/5 and that the y intercept is (0,-3).

- y and x will be kept as a variable.

-plug in the y1 and x1 from the y intercept:

y + 3 = m(x + 0)

-once you've plugged in the y intercept then plug in the slope which gives you the answer:

y + 3= -1/5 x (x+0)

I need help:/ I’m in college

Answers

Step-by-step explanation:

Amount of acid = 14.9% of 331 mL solution

= 0.149×(331 mL)

= 49.3 mL acid

Blood pressure values are often reported to the nearest 5 mmhg (100, 105, 110, etc.). the actual blood pressure values for nine randomly selected individuals are given below.

108.6 117.4 128.4 120.0 103.7 112.0 98.3 121.5 123.2

Required:
a. What is the median of the reported blood pressure values?
b. Suppose the blood pressure of the second individual is 117.7 rather than 117.4 (a small change in a single value). What is the new median of the reported values?
c. What does this say about the sensitivity of the median to rounding or grouping in the data?

Answers

Answer:

Step-by-step explanation:

Arranging the data in the ascending order:

108.6 98.3 103.7 112 117.4 120 121.5 123.2 128.4

The median is the middle value of the data set:

a)

Hence,

median = 117.4

b)

When the value of blood pressure is 117.7 instead of 117.4 then the median will be:

Median = 117.7

c)

This indicates that the median of a well sorted set of data is depends upon the middle value of the data set.

Answer question below

Answers

Answer:

edge of cube = root 13.5/6 = 1.5

volume of cube = (Edge of cube)^3= (1.5)^3 = 3.37500 m^3

what is the completely factored form of this expression?

y^2-12y+32

a.(y+4)(y+8)
b.(y-4)(y-8)
c.(y+18)(y+2)
d.(y-18)(y-2)

Answers

(y-4)(y-8)
Because
-4-8=-12
-4*-8=32

Answer:

[tex]y^2-12y+32[/tex]

[tex]=(x - 4)(x - 8)[/tex]

------------------------

Hope it helps...

Have a great day!!

How do I write this is in a fraction. Please please help
15-(-6) =

Answers

Answer:

5/2

Step-by-step explanation:

You change the signs to positive and then you put 15 as numerator and then 6 as denominator which gives you 15/6. But you can also simplify it which gives you 5/2. Hope this helps!

PLEASE PLEASE HELP ASAPPPP IM BEING TIMEDDD

6x2y − 3xy − 24xy2 + 12y2
Rewrite the expression completely factored. Show the steps of your work.

Answers

Answer:

3y(2x-1)(x-4y)

Step-by-step explanation:

Apply exponent rule:

6x^2y-3xy-24xyy+12yy

Rewrite 12 as 4*3

Rewrite -24 as 8*3

Rewrite 6 as 2*3

2*3x^2y-3xy+8*3xyy+4*3yy

Factor out common term 3y:

3y(2x^2-x-8xy+4y)

Factor 2x^2-x-8xy+4y:

3y(2x-1)(x-4y)

Your Answer Is 3y(2x-1)(x-4y)

is this correct??????

Answers

Answer:

yes, the Temperature is 5 F

Step-by-step explanation:

-3 + 8 = 5

yup that’s correct!! -3+8=5 :D

4. Five cards are randomly chosen from a deck of 52 (13 denominations with 4 suits). a. How many ways are there to receive 5 cards from a deck of 52

Answers

Answer:

There are 2,598,960 ways to receive 5 cards from a deck of 52.

Step-by-step explanation:

The order in which the cards are chosen is not important, which means that the combinations formula is used to solve this question.

Combinations formula:

[tex]C_{n,x}[/tex] is the number of different combinations of x objects from a set of n elements, given by the following formula.

[tex]C_{n,x} = \frac{n!}{x!(n-x)!}[/tex]

a. How many ways are there to receive 5 cards from a deck of 52?

[tex]C_{52,5} = \frac{52!}{5!(47)!} = 2598960[/tex]

There are 2,598,960 ways to receive 5 cards from a deck of 52.

The diameter of the base is the cone measured 8 units. The height measures 6 units.

What is the volume of the cone?

A) 24 π cubic units

B) 32 π cubic units

C)48 π cubic units

D)64 π cubic units

Answers

The answer is B) 32 cubic units

What is the range for the following set of numbers?57, -5, 11, 39, 56, 82, -2, 11, 64, 18, 37, 15, 68

Answers

The range= the highest number - the lowest number

so

82-(-2)

=84

then ur answer is 84

a recent survey shows that 16% of college students have dogs and 38% have an HBO subscription. assuming these two events are independent, what is the probability that a randomly selected college student has neither a dog nor HBO

Answers

Answer: [tex]0.939\ or\ 93.9\%[/tex]

Step-by-step explanation:

Given

Survey shows that 16% of college students have dogs and 38% have HBO subscription

Probability that a random person have both is

[tex]\Rightarrow P_o=0.16\times 0.38\quad [\text{As both events are independent}]\\\Rightarrow P_o=0.0608[/tex]

The probability that the random person has neither of the two is

[tex]\Rightarrow P=1-P_o\\\Rightarrow P=1-0.0608\\\Rightarrow P=0.939[/tex]

If 400 patrons visit the park in March and 550 patrons visit in April, the total number of patrons who
visited the park over the two months falls into all of the following categories except
O real numbers
O rational numbers
o irrational numbers

Answers

950 is a real number, and it is a rational number since it can be expressed as 950/1. Therefore, it falls into all of the categories expect irrational numbers.

Solve and check
y − 5 = −5​

Answers

Answer:

Its -0

Step-by-step explanation:

-0-5=-5

Hope thia helps I believe its right check on a calculator

Help me with this please!!

Answers

Answer:

[tex]y=\sqrt{x} -2[/tex]

Step-by-step explanation:

If you were to replace x with 0 in each of the equations, you would get

from top to bottom choices:

[tex]\sqrt{2}[/tex]

[tex]\sqrt{-2}[/tex]

2

-2

Since we can see that the line touches the y-axis on the number -2, we know that it is the last choice that is the answer

Two planes are the same altitude. From the airport , one plane is 50 km away in the direction of N°60 E and another is 80 km away in the direction of S50° E .How far apart are the two planes

Answers

9514 1404 393

Answer:

  78.5 km

Step-by-step explanation:

Measured at the airport, the angle between the two planes is ...

  180° -60° -50° = 70°

The law of cosines tells us the distance between the planes is ...

  d = √(50² +80² -2·50·80·cos(70°)) ≈ √6163.84 ≈ 78.5 . . . km

The planes are about 78.5 km apart.

Matt buys a new fish tank. The fish tank is in the shape of a cuboid. The diagram shows water in the tank. 30 cm 30 cm 100 cm Matt knows 1000 cm' = 1 litre 1 gallons = 4.5 litres He can keep 2 small fish in the tank for every 1 gallon of water in the tank. Matt thinks he can keep more than 36 small fish in the tank. Is Matt correct?​

Answers

Answer: Yes, but only if he houses 37, 38, 39, or 40 fish

Anything larger than 40 and he'll need more room.

==========================================================

Explanation:

The tank is 30 cm by 30 cm by 100 cm. The volume is 30*30*100 = 90,000 cm^3 which is shorthand for "cubic centimeters".

We're told that 1000 cm^3 = 1 liter, which means the 90,000 cm^3 converts to (90,000)/(1000) = 90 liters.

The fish tank is 90 liters.

Since 1 gallon = 4.5 liters, this means the 90 liter tank converts to 90/(4.5) = 20 gallons

----------------------------

Your teacher mentions "He can keep 2 small fish for every 1 gallon".

Since the tank is 20 gallons, that means he can keep 20*2 = 40 fish. This value is larger than 36, so Matt is correct to a point. If Matt is thinking 37, 38, 39, or 40 fish then he would be correct. If Matt is wanting more than 40 fish, then he'll need a bigger tank.

In short, he can't have any number over 36 and can only have 4 specific values (the four values mentioned earlier).

So technically, Matt is correct, but strong clarification is needed.

Given the triangle below, what is m 2A, rounded to the nearest tenth?
B
18
11
309
A
C
Triangle not drawn to scale
A. 17.8
OB. 20.4
O C. 24.2
D. 26.8

Answers

9514 1404 393

Answer:

  A.  17.8°

Step-by-step explanation:

We have an angle and an opposite side, so we can use the law of sines to find the angle of interest.

  sin(A)/a = sin(C)/c

  A = arcsin(a/c·sin(C)) = arcsin(11/18·1/2) = arcsin(11/36)

  A ≈ 17.7916° ≈ 17.8°

28, 45, 12, 34, 36, 45, 19,20
Part 1: Find the mean of this set of data.
Part 2: Find the mean absolute deviation of this set of data.
Part 3: Using complete sentences, explain what your results for part A and B mean.
i really need an answer for part c

Answers

Part A

To find the mean, we add up the values and divide by n = 8 since there are 8 values in this set.

Adding the values gets us

28+45+12+34+36+45+19+20 = 239

Dividing this over 8 then leads to 239/8 = 29.875

Answer: 29.875

============================================================

Part B

We'll subtract each data value from the mean. We apply absolute value to ensure the result is never negative.

|28 - 29.875| = 1.875 |45 - 29.875| = 15.125 |12 - 29.875| = 17.875 |34 - 29.875| = 4.125 |36 - 29.875| = 6.125 |45 - 29.875| = 15.125 |19 - 29.875| = 10.875 |20 - 29.875| = 9.875

The list of results we get so far is:

1.875, 15.125, 17.875, 4.125, 6.125, 15.125, 10.875, 9.875

This represents the distance each value is from the mean.

Add these values up and divide by n = 8

1.875+15.125+17.875+4.125+6.125+15.125+10.875+9.875 = 81

81/8 = 10.125

Answer: 10.125

============================================================

Part C

The result of part A is one way to measure the center of the distribution of values. It's the average value, which can more or less represent the entire group. Think of it being like how people vote in a senator to represent them in congress. Ideally, this senator is a supposed "average" person to represent everyone.

The result of part B builds on what part A found. The result of part B is the average distance each value is from the center. This is because each time we subtracted and applied absolute value, we found the distance that item was from the mean.

Example: The calculation |28 - 29.875| = 1.875 shows that 28 is exactly 1.875 units from the mean 29.875

By adding up those results and dividing by 8, we are finding the average distance from the mean. Effectively, it tells us how spread out the data set is. The mean absolute deviation (MAD) is a measure of spread in a similar fashion that the standard deviation is, or in a more looser sense, the range is as well.

---------------

In short, the result of part A is a measure of center while the result of part B is a measure of spread. I use "a" instead of "the" because there are other measures of center and other measures of spread.

Hello everyone can someone answer this question please

Answers

9514 1404 393

Answer:

  (a)  2

Step-by-step explanation:

Each inch is 2.54 cm, so 5.08 cm is ...

  x / (5.08 cm) = (1 in) / (2.54 cm)

  x = (1 in)(5.08/2.54) = (1 in)(2)

  x = 2 in

5.08 cm equals 2 inches.

Write the equation of each line in slope intercept form. Slope is -6, and (1,-2) is on the line

Answers

Y = -6x + 4

See the attached photo for further reference.

Hope this helps! Please make me the brainliest, it’s not necessary but appreciated, I put a lot of effort and research into my answers. Have a good day, stay safe and stay healthy.

what is the value of x, given that figure MNOP is a trapezoid with median qr A. 12 B. 8 C. 6 D. 16

Answers

The answer is D. Because if we put 16 in place of x, MP, NO will be 24, 16 respectedly

Step-by-step explanation:

So all horizontal line will be decrease from line MP to No.

As result MP will be = 24, QR = 20, NO = 16. We can see there's different of 4.

The value of x in the trapezoid is 16.

Option D is the correct answer.

What is a trapezium?

It is a quadrilateral that has one pair of parallel sides and a height.

The area is calculated as: 1/2 x sum of the parallel sides x height.

Examples:

Area of a trapezium that has the parallel sides as 3 cm and 4 cm and a heght o 5 cm.

Area = 1/2 x (3 + 4) x 5

Area = 1/2 x 7 x 5

Area = 35/2 = 17.5 cm^2

We have,

In a trapezoid, the median is the average of the parallel sides,

So we have:

QR

= (NO + MP)/2

= (x + x + 8)/2

= (2x + 8)/2

= x + 4

Since we also know that QR = 20, we can set the two expressions equal to each other and solve for x:

x + 4 = 20

x = 16

Therefore,

The value of x is 16.

Learn more about trapezium here:

https://brainly.com/question/22607187

#SPJ7

Other Questions
3x?3x?If Ax)=2xand g(x)find /(x) = g(x) please help! need answers in order to move on:)1.) which number equals (5)^-3?- -125- 1/15- 1/125- -152.) find the equivalent for -(3)^-4- -(4x4x4)- -3x-4- 1/-3x-3x-3x-3- -(1/3x1/3x1/3x1/3)3.) which of the following is equivalent to 3^-8x3^4- 3^-12- 3^-4- 3^-2- 3^-324.) which value is equivalent to 7^-3/7^-5- 7^15- 7^-2- 7^8- 7^25.) choose the equivalent expression (8^10)^2- 8^12- 8^20- 8^8- 8^56.) which expression is equivalent to (9x8)^4- 9x8x4- 9^4x8^4- (9^8)^4- (9^4)^87.) which expression is equivalent to (2/7)^5- 2^5/7^5- 2x5/7x5- 2^5/7- 2x5/7 Which is the graph of y = [x]-2? Please please helped timed 20points Mrs. Turner finally rose to go after being very firm about several other viewpoints of either herself, her son or her brother. She begged Janie to drop in on her anytime, but never once mentioning Tea Cake. Finally she was gone and Janie hurried to her kitchen to put on supper and found Tea Cake sitting in there with his head between his hands. Which best describes the language in this excerpt Find 4^3(5^2). Express using exponents. I NEED HELP ON THIS AS FAST AS POSSIBLE List two important groups currently living in New Mexico. Then, briefly evaluate each groupto identify one way that the a group reflects a historical New Mexican influence. I need the answer of the question without date or percentage thank you guys so much What are the values of a, b, and c in the quadratic equation 0 = one-halfx2 3x 2?a = one-half, b = 3, c = 2a = one-half, b = 3, c = 2a = one-half, b = 3, c = 2a = one-half, b = 3, c = 2 What is the additive identity of -17? Harn una lista de programas de televisin que puedan ayudarnos en algunos de nuestros cursos escolares Larry made 14 baskets out of 21 attempts in a recent basketball game. If Scott attempted 24 baskets and made the same proportion of baskets as Larry, how many baskets did Scott make?Scott made baskets. WHO IS GREATER THAN JHON THE BAPTIST AND WHAT VERCE HELPPPPPP How is the function of the judicial branch most related to the function of the legislative branch?A. The judicial branch proposes new legislation to improve the state and local court systems.B. The judicial branch determines whether the laws passed by Congress are constitutional.C. The judicial branch oversees the impeachment of presidents and other government officials.D.The judicial branch settles disagreements between the Senate and the House of Representatives. A 21-year-old woman presents to the emergency department with fevers, headache, neck stiffness, and mild confusion over the past several days. Her temperature is 38.0 C (100.4 F), pulse 106, and blood pressure 116/74. On physical exam she looks ill, and her neck is stiff. Her neurologic exam is normal. A lumbar puncture reveals 105 WBC and 1240 RBC in tube #1 and 126 WBC and 1360 RBC in tube #4; all white cells are lymphocytes. The CSF protein is 68 and the glucose is 78. This patient most likely has which of the following?1.HSV encephalitis2.Pneumococcal meningitis3.Subarachnoid hemorrhage4.Subdural hematoma I'm under the water on this problem, please help me:question: f(x)=10x3answer choices:A. f(5)B. f(0)C. f(7)D. f(t^2+2)E. f(12x)F. f(x+h) POINTS IF HELP!!! Please help me? Express b+1/3b-2 with b as the subject PLEASE HEEELPPP 15 POINTS PLESSEE Billy jogs 4/5 kilometer every minute.How many kilometers does he jog after 6 1/8 minutes Write a story ending wait that was how I came to know the truthWrite in 3 paragraphs What is the incorrect statement regarding the isotopes of the same element? 1) Electronic configuration is equal 2) Mass number is equal 3) Number of protons are equal 4) Number of electrons are equal